LSAT and Law School Admissions Forum

Get expert LSAT preparation and law school admissions advice from PowerScore Test Preparation.

General questions relating to LSAT Logical Reasoning.
User avatar
 lisamathew21
  • Posts: 1
  • Joined: Apr 16, 2022
|
#94815
Hi all,

Is it acceptable on a Strengthen Q for the correct AC to bolster a premise? Likewise, is it acceptable on a Weaken Q for the correct AC to undermine a premise?

My understanding at this point is that it's okay to undermine a premise on Weaken but not okay to bolster a premise on Strengthen. I just want to get a firm answer on these two questions. Also, the reason why I think you can attack a premise on Weaken Qs is because of PT 75, section 3, question 13, about the University Administrator's opinions on teaching assistants.

Lastly, is there a question type besides Strengthen/Weaken where we can attack premises?

Thank you!
User avatar
 Dave Killoran
PowerScore Staff
  • PowerScore Staff
  • Posts: 5850
  • Joined: Mar 25, 2011
|
#94826
The answer here is yes and yes, although these tend to happen far less than strengthening/attacking the connection between a premise and conclusion.

As to other question types where you weaken a premise, no. That's the only question type in the Third Family :-D

Thanks!

Get the most out of your LSAT Prep Plus subscription.

Analyze and track your performance with our Testing and Analytics Package.